Fourier series and even/odd functions

Click For Summary
The discussion focuses on the application of Fourier series to solve a PDE, specifically examining the function u(rho, phi) expressed as a series of sine and cosine terms. The user questions whether they can discard the A_n coefficients because sin(phi/2) is an odd function, but they learn that A_n must still be calculated. This is due to the integration limits being from 0 to 2π, which do not provide the symmetry required for the A_n terms to vanish. The conversation clarifies the importance of considering the specific interval when determining the contributions of even and odd functions in Fourier series. Understanding this concept is crucial for accurately finding the Fourier coefficients in the given problem.
Niles
Messages
1,834
Reaction score
0
[SOLVED] Fourier series and even/odd functions

Homework Statement


I found the solution to a PDE in this thread: https://www.physicsforums.com/showthread.php?t=224902 (not important)

The solution is the sum of u(rho, phi) = [A_n*cos(n*phi)+B_n*sin(n*phi)]*rho^n.

I have to find the general solution, where rho=c, so I equal rho = c, and I am told that u in this point equals sin(phi/2) when phi is between 0 and 2*pi.

I must find the Fourier-coefficients (since it is a Fourier-series).

My questions are:

Since sin(phi/2) is an ODD function, can I discard A_n and just find B_n? That is what I would do, but in the solutions in the back of my book they find A_n as well. Why is that?! The book even says that for an odd function, the Fourier-series only contains sine, so A_n can be discarded, but they still find it. Can you explain to me why A_n must be found as well?
 
Last edited:
Physics news on Phys.org
The integral of an even function times and odd function will generally vanish only if you are integrating over an interval symmetric around the origin, like [-L,L]. Your interval here is [0,2pi]. The A_n's don't automatically vanish.
 
Ahh, I see.

You have helped me very much lately. Thank you.
 
Question: A clock's minute hand has length 4 and its hour hand has length 3. What is the distance between the tips at the moment when it is increasing most rapidly?(Putnam Exam Question) Answer: Making assumption that both the hands moves at constant angular velocities, the answer is ## \sqrt{7} .## But don't you think this assumption is somewhat doubtful and wrong?

Similar threads

  • · Replies 6 ·
Replies
6
Views
2K
  • · Replies 8 ·
Replies
8
Views
3K
Replies
3
Views
2K
  • · Replies 3 ·
Replies
3
Views
2K
  • · Replies 16 ·
Replies
16
Views
2K
Replies
1
Views
1K
  • · Replies 1 ·
Replies
1
Views
2K
Replies
3
Views
1K
  • · Replies 1 ·
Replies
1
Views
1K
  • · Replies 6 ·
Replies
6
Views
2K